kąty dwuścienne, mnogość oznaczeń

Sześciany. Wielościany. Kule. Inne bryły. Zadania i twierdzenia z nimi związane. Geometria rzutowa w przestrzeni.
wielkireturner
Użytkownik
Użytkownik
Posty: 403
Rejestracja: 8 lut 2015, o 10:46
Płeć: Mężczyzna
Lokalizacja: London ChinaTown
Podziękował: 151 razy
Pomógł: 4 razy

kąty dwuścienne, mnogość oznaczeń

Post autor: wielkireturner »

Dany jest czworościan \(\displaystyle{ A_{1}A_{2}A_{3}A_{4}}\) i punkty \(\displaystyle{ B_{i(i+1)}}\) leżące na krawędziach \(\displaystyle{ A_{i+2}A_{i+3}}\) (dla \(\displaystyle{ i = 1,2,3,4}\) przyjmujemy \(\displaystyle{ A_{k+4} = A_{k}}\)). Każda z płaszczyzn \(\displaystyle{ A_{i}A_{i+1}B_{i(i+1)}}\) tworzy z płaszczyzną \(\displaystyle{ A_{i-1}A_{i}A_{i+1}}\) kąt dwuścienny \(\displaystyle{ \alpha _{i}}\), zaś z płaszczyzną \(\displaystyle{ A_{i}A_{i+1}A_{i+2}}\) kąt dwuścienny o mierze \(\displaystyle{ \beta _{1}}\). Wykazać, że płaszczyzny \(\displaystyle{ A_{i}A_{i+1}B_{i(i+1)}}\) dla \(\displaystyle{ i=1,2,3,4}\) mają wspólny punkt wtedy i tylko wtedy, gdy

\(\displaystyle{ \frac{ \sin \alpha _{1}}{ \sin \beta _{1}} \cdot \frac{ \sin \alpha _{2}}{ \sin \beta _{2}} \cdot \frac{ \sin \alpha _{3}}{ \sin \beta _{3}} \cdot \frac{ \sin \alpha _{4}}{ \sin \beta _{4}} = 1}\)-- 5 sie 2015, o 07:25 --
wielkireturner pisze:Dany jest czworościan \(\displaystyle{ A_{1}A_{2}A_{3}A_{4}}\) i punkty \(\displaystyle{ B_{i(i+1)}}\) leżące na krawędziach \(\displaystyle{ A_{i+2}A_{i+3}}\) (dla \(\displaystyle{ i = 1,2,3,4}\) przyjmujemy \(\displaystyle{ A_{k+4} = A_{k}}\)). Każda z płaszczyzn \(\displaystyle{ A_{i}A_{i+1}B_{i(i+1)}}\) tworzy z płaszczyzną \(\displaystyle{ A_{i-1}A_{i}A_{i+1}}\) kąt dwuścienny \(\displaystyle{ \alpha _{i}}\), zaś z płaszczyzną \(\displaystyle{ A_{i}A_{i+1}A_{i+2}}\) kąt dwuścienny o mierze \(\displaystyle{ \beta _{1}}\). Wykazać, że płaszczyzny \(\displaystyle{ A_{i}A_{i+1}B_{i(i+1)}}\) dla \(\displaystyle{ i=1,2,3,4}\) mają wspólny punkt wtedy i tylko wtedy, gdy

\(\displaystyle{ \frac{ \sin \alpha _{1}}{ \sin \beta _{1}} \cdot \frac{ \sin \alpha _{2}}{ \sin \beta _{2}} \cdot \frac{ \sin \alpha _{3}}{ \sin \beta _{3}} \cdot \frac{ \sin \alpha _{4}}{ \sin \beta _{4}} = 1}\)
edit: Konkretnie nie rozumiem zapisu: \(\displaystyle{ B_{i(i+1)}}\).
ODPOWIEDZ